LSAT and Law School Admissions Forum

Get expert LSAT preparation and law school admissions advice from PowerScore Test Preparation.

User avatar
 Dave Killoran
PowerScore Staff
  • PowerScore Staff
  • Posts: 5850
  • Joined: Mar 25, 2011
|
#40888
Complete Question Explanation
(The complete setup for this game can be found here: lsat/viewtopic.php?t=15551)

The correct answer choice is (B)

If M is second, then from our analysis of the first position, we know that only F, I, and L remain as options to be first. But, I and L are a block, and with M second there is no room for I and L. Thus, F must be first:
PT59_Game_#2_#9_diagram 1.png
Only five variables remain, and those five variables are arranged into two groups: the IL block and the H :longline: GK sequence. Because IL and GK are blocks, H is limited in where it can be placed. H cannot be placed fourth for example, because then there would be no room for the two blocks to fit into the remaining spaces. Because H can never be sixth or seventh from our initial Not Laws, we can infer that H must be either third or fifth. This inference confirms that answer choice (B) cannot be true and is correct (and at the same time eliminates answer choices (A) and (C)).

The two answers that involve G can be eliminated by referring to the two templates created by the placement of H:
PT59_Game_#2_#9_diagram 2.png
 medialaw111516
  • Posts: 80
  • Joined: Dec 11, 2018
|
#71086
Hi,

The way I always handle these is to just go down the answer list and try each option. Is there a better way to do these types of questions?
 Jeremy Press
PowerScore Staff
  • PowerScore Staff
  • Posts: 1000
  • Joined: Jun 12, 2017
|
#71106
Hi medialaw,

The answer-by-answer approach is reliable, but time-consuming. If you want to try to move faster, begin with a strong setup of the local condition and any inferences that results in. Here, as the original post notes, the local condition forces F to be first, and leaves you with the requirement of placing the IL block, plus the H :longline: GK sequence in the spaces between 3 and 7. If you can see that those are the pieces you have to place in that particular space, you can then evaluate (perhaps without diagramming every individual answer) whether you can fit the pieces in. For example, in answer choice A, you can quickly see that placing H third is okay, because the GK block will be after it, and there is plenty of space to "fit" both the IL block and the GK block. That room to "fit" the two blocks disappears in answer choice B, where both the IL and GK blocks would have to be squeezed into spots 5 through 7. Once you see that answer choice B doesn't (and cannot possibly) work, you don't have to spend much time thinking about answers C through E.

I hope this helps!

Jeremy

Get the most out of your LSAT Prep Plus subscription.

Analyze and track your performance with our Testing and Analytics Package.